TRIGONOMETRY - 2 (Trigonometric Functions)

1. $\sin 47^{\circ}+\sin 61^{\circ}-\sin 11^{\circ}-\sin 25^{\circ}$ is equal to

(a) $\sin 36^{\circ}$

(b) $\cos 36^{\circ}$

(c) $\sin 7^{\circ}$

(d) $\cos 7^{\circ}$

Show Answer

Solution:

$ \begin{aligned} & \sin 47+\sin 61^{\circ}-\sin 11^{\circ}-\sin 25^{\circ} \\ & \left(2 \sin 54^{\circ} \cos 7^{\circ}\right)-\left(2 \sin 18^{\circ} \cos 7^{\circ}\right) \\ & =2 \cos 7^{\circ}\left(\sin 54^{\circ}-\sin 18^{\circ}\right) \\ & =2 \cos 7^{\circ}\left(\cos 36^{\circ}-\sin 18^{\circ}\right) \\ & =2 \cos 7^{\circ}\left(\frac{\sqrt{5}+1}{4}-\left(\frac{\sqrt{5}-1}{4}\right)\right) \\ & =2 \cos 7^{\circ}\left(\frac{1}{2}\right) \\ & =\cos 7^{\circ} \end{aligned} $

Correct option is ’d ‘

2. If $\frac{\mathrm{x}}{\cos \theta}=\frac{\mathrm{y}}{\cos \left(\theta-\frac{2 \pi}{3}\right)}=\frac{\mathrm{z}}{\cos \left(\theta+\frac{2 \pi}{3}\right)}$, then $\mathrm{x}+\mathrm{y}+\mathrm{z}$ is equal to

(a) $1$

(b) $0$

(c) $-1$

(d) none of these

Show Answer

Solution:

We have

$ \frac{\mathrm{x}}{\cos \theta}=\frac{\mathrm{y}}{\cos \left(\theta-\frac{2 \pi}{3}\right)}=\frac{\mathrm{z}}{\cos \left(\theta+\frac{2 \pi}{3}\right)} $

we know that each ratio is equal to $\frac{\text { sum of Numerator }}{\text { sum of denominator }}$

$\therefore \frac{\mathrm{x}}{\cos \theta}=\frac{\mathrm{y}}{\cos \left(\theta-\frac{2 \pi}{3}\right)}=\frac{\mathrm{z}}{\cos \left(\theta+\frac{2 \pi}{3}\right)}=\frac{\mathrm{x}+\mathrm{y}+\mathrm{z}}{\cos \theta+\cos \left(\theta-\frac{2 \pi}{3}\right)+\cos \left(\theta+\frac{2 \pi}{3}\right)}$

now, $\quad \cos \theta+\cos \left(\theta-\frac{2 \pi}{3}\right)+\cos \left(\theta+\frac{2 \pi}{3}\right)$

$=\cos \theta+2 \cos \theta \cos \frac{2 \pi}{3} \quad(\cos (\mathrm{A}+\mathrm{B})+\cos (\mathrm{A}-\mathrm{B})=2 \cos \mathrm{A} \cos \mathrm{B})$

$=\cos \theta+2 \cos \theta\left(-\frac{1}{2}\right)$

$=0$

$\therefore \mathrm{x}+\mathrm{y}+\mathrm{z}=0$

Correct option is: ‘b’

3. The value of $\left(1+\cos \frac{\pi}{8}\right)\left(1+\cos \frac{3 \pi}{8}\right)\left(1+\cos \frac{5 \pi}{8}\right)\left(1+\cos \frac{7 \pi}{8}\right)$ is

(a) $\frac{1}{2}$

(b) $\cos \frac{\pi}{8}$

(c) $\frac{1}{8}$

(d) $\frac{1+\sqrt{2}}{2 \sqrt{2}}$

Show Answer

Solution:

$ \begin{aligned} & \cos \frac{7 \pi}{8}=\cos \left(\pi-\frac{\pi}{8}\right)=-\cos \frac{\pi}{8} \\ & \cos \frac{3 \pi}{8}=\cos \left(\frac{\pi}{2}-\frac{\pi}{8}\right)=\sin \frac{\pi}{8} \\ & \cos \frac{5 \pi}{8}=\cos \left(\frac{\pi}{2}+\frac{\pi}{8}\right)=-\sin \frac{\pi}{8} \\ & =\left(1+\cos \frac{\pi}{8}\right)\left(1+\sin \frac{\pi}{8}\right)\left(1-\sin \frac{\pi}{8}\right)\left(1-\cos \frac{\pi}{8}\right) \\ & =\left(1-\cos ^{2} \frac{\pi}{8}\right)\left(1-\sin ^{2} \frac{\pi}{8}\right) \\ & =\sin ^{2} \frac{\pi}{8} \cos ^{2} \frac{\pi}{8} \\ & =\frac{\left(2 \sin \frac{\pi}{8} \cos \frac{\pi}{8}\right)^{2}}{4}=\frac{\left(\sin \frac{\pi}{4}\right)^{2}}{4}=\frac{1}{8} \end{aligned} $

Correct option is: ‘c’

4. If $\mathrm{x} _{1}, \mathrm{x} _{2} \ldots \mathrm{x} _{\mathrm{n}}$ are in AP whose common difference is $\alpha$, then the value of $\sin \alpha\left(\operatorname{\sec x} _{1} \operatorname{\sec x} _{2}+\operatorname{\sec x} _{2} \sec _{3}+\ldots \ldots . .+\operatorname{\sec x} _{n-1} \sec _{n}\right)$ is

(a) $\frac{\sin (\mathrm{n}-1) \alpha}{\cos \mathrm{x} _{1} \cos \mathrm{x} _{\mathrm{n}}}$

(b) $\frac{\sin n \alpha}{\cos x _{1} \cos x _{n}}$

(c) $\sin (\mathrm{n}-1) \alpha \cos \mathrm{x} _{1} \cos \mathrm{x} _{\mathrm{n}}$

(d) $\sin \alpha \cos x _{1} \cos x _{n}$

Show Answer

Solution:

We have

$\sin \alpha \sec x _{1} \sec _{2}+\sin \alpha \sec x _{2} \sec x _{3}+\ldots \ldots \ldots \ldots+\sin \alpha \sec x _{n-1} \sec x _{n}$

$=\frac{\sin \left(\mathrm{x} _{2}-\mathrm{x} _{1}\right)}{\cos \mathrm{x} _{1} \cos \mathrm{x} _{2}}+\frac{\sin \left(\mathrm{x} _{3}-\mathrm{x} _{2}\right)}{\cos \mathrm{x} _{2} \cos \mathrm{x} _{3}}+\ldots \ldots \ldots+\frac{\sin \left(\mathrm{x} _{\mathrm{n}}-\mathrm{x} _{\mathrm{n}-1}\right)}{\cos \mathrm{x} _{\mathrm{n}-1} \cos \mathrm{x} _{\mathrm{n}}}$

$=\frac{\sin x _{2} \cos x _{1}-\cos x _{2} \sin x _{1}}{\cos x _{1} \cos x _{2}}+\frac{\sin x _{3} \cos x _{2}-\cos x _{3} \sin x _{2}}{\cos x _{2} \cos x _{3}}+$

$+\frac{\sin x _{n} \cos x _{n-1}-\cos x _{n} \sin x _{n-1}}{\cos x _{n-1} \cos x _{n}}$

$=\tan x _{2}-\tan x _{1}+\tan x _{3}-\tan x _{2}+\ldots \ldots .+\tan x _{n}-\tan x _{n-1}$

$=\tan x _{n}-\tan x _{1}$

$=\frac{\sin x _{n} \cos x _{1}-\cos x _{n} \sin x _{1}}{\cos x _{n} \cos x _{1}}$

$=\frac{\sin \left(\mathrm{x} _{\mathrm{n}}-\mathrm{x} _{1}\right)}{\cos \mathrm{x} _{\mathrm{n}} \cos \mathrm{x} _{1}}$

$=\frac{\sin (\mathrm{n}-1) \alpha}{\cos \mathrm{x} _{\mathrm{n}} \cos \mathrm{x} _{1}} \quad\left(\because \mathrm{x} _{\mathrm{n}}=\mathrm{x} _{1}+(\mathrm{n}-1) \alpha \quad \mathrm{x} _{\mathrm{n}}-\mathrm{x} _{1}=(\mathrm{n}-1) \alpha\right)$

$\therefore$ correct option is ‘a’

5. Let $f(\mathrm{n})=2 \operatorname{cosnx} \forall \mathrm{n} \in \mathrm{N}$, then $f(1) f(\mathrm{n}+1)-f(\mathrm{n})$ is equal to

(a) $\mathrm{f}(\mathrm{n}+3)$

(b) $\mathrm{f}(\mathrm{n}+2)$

(c) $\mathrm{f}(\mathrm{n}+1) \mathrm{f}(2)$

(d) $\mathrm{f}(\mathrm{n}+2) \mathrm{f}(2)$

Show Answer

Solution:

We have $\mathrm{f}(\mathrm{n})=2 \operatorname{cosnx} \forall \mathrm{n} \in \mathrm{N}$

$\mathrm{f}(1)=2 \cos \mathrm{x}$

$\mathrm{f}(\mathrm{n}+1)=2 \cos (\mathrm{n}+1) \mathrm{x}$

$f(1) f(n+1)=4 \cos x \cos (n+1) x$

$\mathrm{f}(1) \mathrm{f}(\mathrm{n}+1)-\mathrm{f}(\mathrm{n})=4 \cos \mathrm{x} \cos (\mathrm{n}+1) \mathrm{x}-2 \cos n \mathrm{x}$

$=2[2 \cos x \cos (n+1) x-\cos n x]$

$=2[\cos (n+2) x+\cos n x-\cos n x]$

$=2 \cos (\mathrm{n}+2) \mathrm{x}$

$=\mathrm{f}(\mathrm{n}+2)$

Option ‘b’ is correct

6. The ratio of the greatest value of $2-\cos x+\sin ^{2} x$ to its least value is

(a) $1 / 4$

(b) $9 / 4$

(c) $13 / 4$

(d) None of these

Show Answer

Solution:

$2-\cos x+\sin ^{2} x$

$=2-\cos x+1-\cos ^{2} x$

$=-\left(\cos ^{2} x+\cos x\right)+3$

$=-\left(\cos x+\frac{1}{2}\right)^{2}+\frac{1}{4}+3$

$=-\left(\cos x+\frac{1}{2}\right)^{2}+\frac{13}{4}$

$ =\frac{13}{4}-\left(\cos x+\frac{1}{2}\right)^{2} $

Maximum value $\frac{13}{4}$ occurs at $\cos x=-\frac{1}{2}$

Minimum value occurs at $\cos x=1$

$ \begin{aligned} & \therefore \frac{13}{4}-\left(1+\frac{1}{2}\right)^{2} \\ & \frac{13}{4}-\frac{9}{4}=\frac{4}{4}=1 \end{aligned} $

$\therefore$ The ratio of greatest to the least is $\frac{13}{4}$

Option ‘c’ is correct

7. If $\sin (120-\alpha)=\sin (120-\beta), 0<\alpha, \beta<\pi$, then find the relation between $\alpha$ and $\beta$.

Show Answer

Solution:

$ \begin{aligned} & \text { If } \sin A=\sin B \\ & \text { Then } A=B \text { or } A=\pi-B \\ & \text { Here, } \sin \left(120^{\circ} \alpha\right)=\sin \left(120^{\circ}-\beta\right) \\ & \therefore 120^{\circ} \alpha=120^{\circ}-\beta \text { or } 120^{\circ} \alpha=\pi-\left(120^{\circ} \beta\right) \\ & \alpha=\beta \text { or } 120^{\circ}-\alpha=60^{\circ}+\beta \alpha+\beta=60^{\circ} \end{aligned} $

8. If $x, y, z$ are in $A P$, then $\frac{\sin x-\sin z}{\cos z-\cos x}$ is equal to

(a) $\tan y$

(b) $\cot y$

(c) $\sin y$

(d) $\cos y$

Show Answer

Solution:

We have $\mathrm{x}, \mathrm{y}, \mathrm{z}$ are in $\mathrm{AP}$

$\therefore \mathrm{x}+\mathrm{z}=2 \mathrm{y}$

Here, $\frac{\sin x-\sin z}{\cos z-\cos x}=\frac{2 \cos \frac{x+z}{2} \sin \left(\frac{x-z}{2}\right)}{2 \sin \left(\frac{x+z}{2}\right) \sin \left(\frac{x-z}{2}\right)}$

$ \begin{aligned} & =\frac{\cos y}{\sin y} \\ & =\operatorname{\cot y} \end{aligned} $

Correct option is ‘b’

Practice questions

1. It $\cos \theta=\frac{a \cos \phi+b}{a+b \cos \phi}$, then $\tan \theta / 2$ equals

(a). $\sqrt{\frac{a-b}{a+b}} \tan \frac{\phi}{2}$

(b). $\sqrt{\frac{a+b}{a-b}} \cos \frac{\phi}{2}$

(c). $\sqrt{\frac{a-b}{a+b}} \sin \frac{\phi}{2}$

(d). none

Show Answer Answer: (a)

2. If $a \cos 2 \theta+b \sin 2 \theta=c$ has $\alpha$ and $\beta$ as its solution, then $\tan \alpha+\tan \beta$ equals

(a). $\frac{2 a}{b+c}$

(b). $\frac{2 b}{c+a}$

(c). $\frac{2 c}{a+b}$

(d). none

Show Answer Answer: (b, c)

3. The value of $\frac{1}{\sin 10^{0}}-\frac{\sqrt{3}}{\cos 10^{\circ}}$ equals

(a). 1

(b). 4

(c). 2

(d). 0

Show Answer Answer: (b)

4. $\cos ^{4} \frac{\pi}{8}+\cos ^{4} \frac{3 \pi}{8}+\cos ^{4} \frac{5 \pi}{8}+\cos ^{4} \frac{7 \pi}{8}$ equals

(a). $\frac{1}{2}$

(b). $1$

(c). $\frac{3}{2}$

(d). $2$

Show Answer Answer: (c)

5. $\sin \frac{\pi}{n}+\sin \frac{3 \pi}{n}+\sin \frac{5 \pi}{n}+…….n$ terms equals

(a). $1$

(b). $0$

(c). $\frac{\mathrm{n}}{2}$

(d). none

Show Answer Answer: (b)

6. If $x=\frac{\sqrt{1-\sin 4 \theta}+1}{\sqrt{1+\sin 4 \theta}-1}$ then one of the values of $x$ is

(a). $-\tan \theta$

(b). $\cot \theta$

(c). $\tan (\pi / 4+\theta)$

(d). $-\cot (\pi / 4+\theta)$

Show Answer Answer: (a, b, c, d)

7. The value of $\cos 12^{\circ}+\cos 84^{\circ}+\cos 156^{\circ}+\cos 132^{\circ}$ is

(a). $\frac{1}{2}$

(b). $1$

(c). $-\frac{1}{2}$

(d). $\frac{1}{8}$

Show Answer Answer: (c)

8. $\sin 6^{\circ}-\sin 66^{\circ}+\sin 78^{\circ}-\sin 42^{\circ}$ is

(a). $-1$

(b). $-\frac{1}{2}$

(c). $\frac{1}{2}$

(d). $1$

Show Answer Answer: (b)

9. $\cos \frac{\pi}{65} \cos \frac{2 \pi}{65} \cos \frac{4 \pi}{65} \cos \frac{8 \pi}{65} \cos \frac{16 \pi}{65} \cos \frac{32 \pi}{65}$ equals

(a). $\frac{1}{8}$

(b). $\frac{1}{16}$

(c). $\frac{1}{32}$

(d). $\frac{1}{64}$

Show Answer Answer: (d)

10. $\sin 36^{\circ} \sin 72^{\circ} \sin 108^{\circ} \sin 144^{\circ}$ equals

(a). $\frac{5}{16}$

(b). $\frac{3}{16}$

(c). $\frac{1}{16}$

(d). none

Show Answer Answer: (a)

11. Passage

If $\mathrm{A}, \mathrm{B}, \mathrm{C}$ be the angles of a triangle, then

(a). $\quad \sum \sin 2 \mathrm{~A}=4 \sin \mathrm{A} \sin \mathrm{B} \sin \mathrm{C}$

(b). $\quad \sum \cos 2 \mathrm{~A}=1-4 \cos \mathrm{A} \cos \mathrm{B} \cos \mathrm{C}$

(c). $\quad \sum \sin \mathrm{A}=4 \cos \mathrm{A} / 2 \cos \mathrm{B} / 2 \cos \mathrm{C} / 2$

(d). $\quad \sum \cos \mathrm{A}=-1+4 \sin \mathrm{A} / 2 \sin \mathrm{B} / 2 \sin \mathrm{C} / 2$

(e). $\quad \sum \tan \mathrm{A}=\tan \mathrm{A} \tan \mathrm{B} \tan \mathrm{C}$ ie., $\mathrm{S} _{1}=\mathrm{S} _{3}$

(f). $\quad \sum \tan \mathrm{A} / 2 \tan \mathrm{B} / 2=1$ or $\sum \cot \mathrm{A} / 2=\cot \mathrm{A} / 2 \cot \mathrm{B} / 2 \cot \mathrm{C} / 2$

Answer the following questions based upon above passage.

i. In a triangle $A B C \frac{\sin A+\sin B+\sin C}{\sin A+\sin B-\sin C}$ equals

(a). $\tan \mathrm{A} / 2 \cot \mathrm{B} / 2$

(b). $\cot \mathrm{A} / 2 \tan \mathrm{B} / 2$

(c). $\cot \mathrm{A} / 2 \cot \mathrm{B} / 2$

(d). $\tan \mathrm{A} / 2 \tan \mathrm{B} / 2$

Show Answer Answer: (c)

ii. $\sin ^{2} \mathrm{~A}+\sin ^{2} \mathrm{~B}+\sin ^{2} \mathrm{C}-2 \cos \mathrm{A} \cos \mathrm{B} \cos \mathrm{C}$ equals

(a). 1

(b). 2

(c). 3

(d). 4

Show Answer Answer: (b)

iii. In a $\triangle \mathrm{ABC}$, whose angles are acute and positive such that $\mathrm{A}+\mathrm{B}+\mathrm{C}=\pi$ and $\cot \mathrm{A} / 2 \cot \mathrm{B} / 2 \cot \mathrm{C} / 2=\mathrm{K}$ then

(a). $\mathrm{K} \leq 3$

(b). $\mathrm{K} \leq 3 \sqrt{3}$

(c). $\mathrm{K} \geq 3 \sqrt{3}$

(d). none

Show Answer Answer: (c)

iv. $\tan \mathrm{A}, \tan \mathrm{B}, \tan C$ are the roots of the cubic equation $\mathrm{x}^{3}-7 \mathrm{x}^{2}+11 \mathrm{x}-7=0$ then $\mathrm{A}+\mathrm{B}+\mathrm{C}$ equals

(a). $\frac{\pi}{2}$

(b). $\pi$

(c). $0$

(d). none

Show Answer Answer: (b)

12. Matchning type question :-

Column I Column II
a. $\sin (\mathrm{B}+\mathrm{C}-\mathrm{A})+\sin (\mathrm{C}+\mathrm{A}-\mathrm{B})+$ $\sin (\mathrm{A}+\mathrm{B}-\mathrm{C})=$ p. $1-2\cos\frac{A}{2}\cos \frac{B}{2}\cos\frac{C}{2}$
b. $\sin ^{2} \frac{\mathrm{A}}{2}+\sin ^{2} \frac{\mathrm{B}}{2}-\sin ^{2} \frac{\mathrm{C}}{2}=$ q. $1$
c. If $\tan ^{2} \frac{\mathrm{A}}{2}+\tan ^{2} \frac{\mathrm{B}}{2}+\tan ^{2} \frac{\mathrm{C}}{2}=\mathrm{K}$ r. $4 \sin \mathrm{A} \sin \mathrm{B} \sin \mathrm{C}$

then $\mathrm{K} \geq$

Show Answer Answer: a - r, b - p, c - q

Assertion and reason type questions

$\mathrm{A}$ : Both $\mathrm{A}$ and $\mathrm{R}$ are individually true and $\mathrm{R}$ is the correct explaination of $\mathrm{A}$

$\mathrm{B}$ : Both $\mathrm{A}$ and $\mathrm{R}$ are individually true and $\mathrm{R}$ is not the correct explaination of $\mathrm{A}$

$\mathrm{C}: \mathrm{A}$ is true but $\mathrm{R}$ is false

$\mathrm{D}: \mathrm{A}$ is false but $\mathrm{R}$ is true

13. Assertion (A) : $\tan \theta+2 \tan 2 \theta+4 \tan 4 \theta+8 \tan 8 \theta+16 \cot 16 \theta=\cot \theta$

Reason $(\mathrm{R}): \cot \theta-\tan \theta=2 \cot 2 \theta$

(a). A

(b). B

(c). C

(d). D

Show Answer Answer: (a)

14. Assertion (A): $\sec ^{2} \theta=\frac{4 x y}{(x+y)^{2}} \Rightarrow x=y$

Reason $(\mathrm{R}): \sec \theta \geq 1$

(a). A

(b). B

(c). C

(d). D

Show Answer Answer: (a)

15. Assertion (A) : If A, B, C, D be the angles of a cyclic quadrilateral then $\cos \mathrm{A}+\cos \mathrm{B}+\cos \mathrm{C}+\cos \mathrm{D}=0$

Reason $(\mathrm{R}): \sin \mathrm{A}+\sin \mathrm{B}+\sin \mathrm{C}+\sin \mathrm{D}=0$

(a). A

(b). B

(c). C

(d). D

Show Answer Answer: (c)


Table of Contents